Difference between revisions of "2016 AMC 12B Problems/Problem 24"

m (Problem)
Line 5: Line 5:
  
 
=Solution=
 
=Solution=
 +
{{solution}}
 +
 +
==See Also==
 +
{{AMC12 box|year=2016|ab=B|num-a=25|num-b=23}}

Revision as of 18:04, 21 February 2016

Problem

There are exactly $77,000$ ordered quadruplets $(a, b, c, d)$ such that $GCD(a, b, c, d) = 77$ and $LCM(a, b, c, d) = n$. What is the smallest possible value for $n$?

$\textbf{(A)}\ 13,860\qquad\textbf{(B)}\ 20,790\qquad\textbf{(C)}\ 21,560 \qquad\textbf{(D)} 27,720 \qquad\textbf{(E)}\ 41,580$

Solution

This problem needs a solution. If you have a solution for it, please help us out by adding it.

See Also

2016 AMC 12B (ProblemsAnswer KeyResources)
Preceded by
Problem 23
Followed by
Problem 25
1 2 3 4 5 6 7 8 9 10 11 12 13 14 15 16 17 18 19 20 21 22 23 24 25
All AMC 12 Problems and Solutions